ChaseDream
搜索
返回列表 发新帖
00:00:00

Lightbox, Inc., owns almost all of the movie theaters in Washington County and has announced plans to double the number of movie screens it has in the county within five years. Yet attendance at Lightbox's theaters is only just large enough for profitability now and the county's population is not expected to increase over the next ten years. Clearly, therefore, if there is indeed no increase in population, Lightbox's new screens are unlikely to prove profitable.

Which of the following, if true about Washington County, most seriously weakens the argument?

正确答案: A

更多相关帖子

524

帖子

15

好友

4712

积分

ChaseDream

注册时间
2003-03-17
精华
8
解析
查看: 2152|回复: 5
打印 上一主题 下一主题

请教GWD-5-15,讨论中的没说明白

[复制链接]
楼主
发表于 2005-9-9 15:54:00 | 只看该作者

请教GWD-5-15,讨论中的没说明白

Q15:


Lightbox, Inc., owns almost all of the movie theaters in Washington County and has announced plans to double the number of movie screens it has in the county within five years.  Yet attendance at Lightbox’s theaters is only just large enough for profitability now and the county’s population is not expected to increase over the next ten years.  Clearly, therefore, if there is indeed no increase in population, Lightbox’s new screens are unlikely to prove profitable.



Which of the following, if true about Washington County, most seriously weakens the argument?




  • Though little change in the size of the population is expected, a pronounced shift toward a younger, more affluent, and more entertainment-oriented population is expected to occur.

  • The sales of snacks and drinks in its movie theaters account for more of Lightbox’s profits than ticket sales do.

  • In selecting the mix of movies shown at its theaters, Lightbox’s policy is to avoid those that appeal to only a small segment of the moviegoing population.

  • Spending on video purchases, as well as spending on video rentals, is currently no longer increasing.

  • There are no population centers in the county that are not already served by at least one of the movie theaters that Lightbox owns and operates.

  • 我选了C,但其实我觉得哪个都不太对,请各位帮忙 谢拉

    沙发
    发表于 2005-9-9 18:01:00 | 只看该作者

    这道题如果我没记错应该是选A吧


    conclusion: Clearly, therefore, if there is indeed no increase in population, Lightbox’s new screens are unlikely to prove profitable.


    a.Though little change in the size of the population is expected, a pronounced shift toward a younger, more affluent, and more entertainment-oriented population is expected to occur.


    意思是说虽然人口总数没有太大变化,但由于人口年龄结构的变化有更多人对娱乐感兴趣,也就会有更多人到电影院,所以削弱了结论。(个人意见)

    板凳
     楼主| 发表于 2005-9-9 18:40:00 | 只看该作者

    谢谢,有一定道理,但younger, more affluent, and more entertainment-oriented population就一定会去电影院从而prove profitable吗?


    如果收入不抵成本呢?


    [此贴子已经被作者于2005-9-9 19:01:39编辑过]
    地板
    发表于 2005-9-9 20:01:00 | 只看该作者
    我觉得这可以用断桥来说明,这个if条件句暗含了一个假设就是说去看电影的人在总人口的比重是不变的,而A恰好否定了这个假设,所以是削弱。
    5#
     楼主| 发表于 2005-9-10 05:57:00 | 只看该作者

    make sense!


    thanks a lot 天之角

    6#
    发表于 2008-7-9 12:59:00 | 只看该作者
    以下是引用天之角在2005-9-9 20:01:00的发言:
    我觉得这可以用断桥来说明,这个if条件句暗含了一个假设就是说去看电影的人在总人口的比重是不变的,而A恰好否定了这个假设,所以是削弱。

    我觉得C也可以否定这个假设亚。。。不明白

    您需要登录后才可以回帖 登录 | 立即注册

    Mark一下! 看一下! 顶楼主! 感谢分享! 快速回复:

    手机版|ChaseDream|GMT+8, 2024-4-28 13:28
    京公网安备11010202008513号 京ICP证101109号 京ICP备12012021号

    ChaseDream 论坛

    © 2003-2023 ChaseDream.com. All Rights Reserved.

    返回顶部